¿Calcular el número de cuantos emitidos como radiación de Cherenkov en un intervalo de longitud de onda dl?

Este sitio web da una ecuación para ello http://math.ucr.edu/home/baez/physics/Relativity/SpeedOfLight/cherenkov.html

Aparece como texto sin formato, pero esto es lo que creo que se analiza: norte = d yo 2 π α L pecado 2 θ yo 2

Y este es el texto que rodea y sobre la ecuación:

Para la mayoría de los medios, la luz azul predomina sobre las longitudes de onda de luz más largas, porque el número de cuantos emitidos como radiación de Cherenkov en un intervalo de longitud de onda dl en una longitud de onda l sobre una longitud de trayectoria L viene dado por:

donde α es la constante de estructura fina, igual a aproximadamente 1/137. Observe que el índice de refracción y, por lo tanto, el ángulo θ, cambia con la longitud de onda l, como se demuestra cuando un prisma produce un espectro de luz blanca. Esto suprime la tasa en las pequeñas longitudes de onda del ultravioleta y más allá.

Otros sitios web enumeran esta fórmula pero parecen haber copiado y pegado el texto que la rodea.

No es una ecuación en la forma dada, pero asumo por el texto que la rodea que una norte = podría agregarse para el número de cuantos.

Tres preguntas: ¿He interpretado correctamente esta ecuación?

Si es así, ¿por qué no cancelaron el factor común de yo .

¿Y alguien puede vincular o mostrar una derivación de esta ecuación o cualquiera que sea la ecuación correcta?

Esta fórmula parece demasiado simple a primera vista. La fórmula actual debería verse así en.wikipedia.org/wiki/Frank%E2%80%93Tamm_formula y la parte más complicada es obtener la función correcta del índice de refracción del material para la región ultravioleta profunda. También vea mi pregunta physics.stackexchange.com/questions/72446/…
@BarsMonster Sí, pensé que podría ser una aproximación, pero aún útil. La fórmula de Frank-Tamm da valores para la energía pero no para el número de fotones liberados a una determinada longitud de onda, que es lo que pretende encontrar esta ecuación.

Respuestas (1)

De ninguna manera soy un experto en esto, pero como esta pregunta me pareció interesante, busqué una pista en la inmensidad infinita de Internet y encontré tres enlaces:

¿He interpretado correctamente esta ecuación?

Básicamente sí. Debes escribirlo de la siguiente manera:

norte = 2 π α Z 2 L pecado 2 θ d λ λ 2

(para que no haya confusión entre la longitud L y la longitud de onda lambda)

En los enlaces que escriben d norte d X , por lo que cuando lo escribe de la forma en que lo hizo, ya se ha integrado sobre dx. Pero aún tiene que integrarse en el rango de longitudes de onda de los fotones emitidos, es por eso que no puede cancelar el d λ -factor.

Desde porque ( θ ) = 1 norte β , puedes escribir el s i norte ( θ ) 2 como:

( 1 1 β 2 norte 2 ) que luego da

d norte d X = 2 π α Z 2 d λ λ 2 ( 1 1 β 2 norte 2 )
como se indica en los dos segundos enlaces.

editar: me acabo de dar cuenta de que ignoré por completo que no hay Z en su fórmula. Entonces Z es la carga de la partícula que viaja a través del medio. Cuando desea calcular la cantidad de fotones emitidos por un electrón, entonces Z obviamente sería 1.